Monday, December 23, 2013

Assumptions and Causality


The central assumption of causality was stated in the last chapter:
“When an LSAT speaker concludes that one occurrence caused another,
that speaker also assumes that the stated cause is the only possible cause
of the effect and that consequently the stated cause will always produce
the effect.”
Thus, because the author always assumes that the stated cause is the only cause,
Assumption answer choices tend to work exactly like Strengthen answer
choices in arguments with causal reasoning. The correct answer to an
Assumption question will normally fit one of the following categories:
A. Eliminates any alternate cause for the stated effect
Because the author believes there is only one cause (the stated cause in
the argument), the author assumes no other cause exists.
B. Shows that when the cause occurs, the effect occurs
Because the author believes that the cause always produces the effect,
assumption answers will affirm this relationship.
C. Shows that when the cause does not occur, the effect does not occur
Using the reasoning in the previous point, the author will always assume
that when the cause does not occur, the effect will not occur.
D. Eliminates the possibility that the stated relationship is reversed
Because the author believes that the cause-and-effect relationship is
correctly stated, the author assumes that the relationship cannot be
backwards (the claimed effect is actually the cause of the claimed
cause).
E. Shows that the data used to make the causal statement are accurate, or
eliminates possible problems with the data
If the data used to make a causal statement are in error, then the validity
of the causal claim is in question. The author assumes that this cannot be
the case and that the data are accurate.
The above categories should be easy to identify because you should have
already memorized them from the Strengthen question section. From now on,
when you encounter Assumption questions containing causal reasoning, you
will be amazed at how obvious the correct answer will seem. These types of
patterns within questions are what makes improvement on the LSAT possible,
and when you become comfortable with the ideas, your speed will also
increase.
Please take a moment to complete the following problem:
4. Doctors in Britain have long suspected that patients
who wear tinted eyeglasses are abnormally prone to
depression and hypochondria. Psychological tests
given there to hospital patients admitted for physical
complaints like heart pain and digestive distress
confirmed such a relationship. Perhaps people whose
relationship to the world is psychologically painful
choose such glasses to reduce visual stimulation,
which is perceived as irritating. At any rate, it can be
concluded that when such glasses are worn, it is
because the wearer has a tendency to be depressed or
hypochondriacal.
The argument assumes which one of the following?
(A) Depression is not caused in some cases by an
organic condition of the body.
(B) Wearers do not think of the tinted glasses as a
means of distancing themselves from other
people.
(C) Depression can have many causes, including
actual conditions about which it is
reasonable for anyone to be depressed.
(D) For hypochondriacs wearing tinted glasses, the
glasses serve as a visual signal to others that
the wearer’s health is delicate.
(E) The tinting does not dim light to the eye
enough to depress the wearer’s mood
substantially.
The conclusion of this argument is causal in nature (“because” is the indicator):
Depression = tendency to be depressed or hypochondriacal
Glasses = glasses are worn
C                   E
Depression-------> Glasses
The answer choices are very interesting as they all relate to either the cause or
effect, or both. Answer choices (A) and (C) are similar in that they both discuss
what causes depression (the cause of the cause). But the author has made no
assumption about what causes depression, only that depression causes a person
to wear glasses. Therefore, both of these answers are incorrect. Similarly,
answer choices (B) and (D) both discuss the effects of wearing glasses (the
effects of the effect). Again, this is not a part of the author’s argument. Because
answer choices (A), (B), (C), and (D) discuss issues that occur either “before”
or “after” the causal relationship in the conclusion, they are incorrect.
Answer choice (E): This is the correct answer. Answer choice (E) is a Defender
that eliminates the possibility that the stated relationship is reversed (Type D in
the Assumptions and Causality discussion). Remember, if the glasses actually
cause the wearer to be depressed, this scenario would hurt the argument, so the
author assumes the possibility cannot exist. Note how tricky this answer could
be, especially if you had not been exposed to the way the test makers think
about causality and assumptions. With the right information, the answer can be
identified as part of a larger pattern on the LSAT, and this allows you to solve
the problem quickly and confidently. While it may take a bit of work to
memorize the different assumptions inherent in causal arguments, the payoff is
more than worth the effort.

No comments:

Post a Comment